Классы BPP — различия между версиями

Материал из Викиконспекты
Перейти к: навигация, поиск
(Новая страница: «==Определения== {{Определение |definition= <tex>BPP_{weak}</tex> — множество языков <tex>L</tex>, для которых с...»)
 
(кат)
(не показано 29 промежуточных версий 7 участников)
Строка 1: Строка 1:
 
==Определения==
 
==Определения==
 +
{{Определение
 +
|definition =
 +
<tex>\mathrm{BPP}</tex> (от ''bounded probabilistic polynomial'') — множество языков <tex>L</tex>, для которых существует такая [[Вероятностные вычисления. Вероятностная машина Тьюринга |ВМТ]] <tex>p</tex>, что для любого <tex>x</tex>:
 +
# <tex>P(p(x) = [x \in L]) \ge 2/3</tex>;
 +
# <tex>T(p, x) \le poly(|x|)</tex> для любой [[Вероятностные вычисления. Вероятностная машина Тьюринга |вероятностной ленты]].
 +
}}
 +
 +
<tex>\mathrm{BPP}</tex> — сложностный класс, допускающий двусторонние ошибки.
 +
Константу <tex>2/3</tex> можно заменить на любое число из промежутка <tex>(1/2, 1)</tex>, так как требуемой вероятности можно добиться множественным запуском <tex>p</tex>. Замена константы на <tex>1/2</tex> сделала бы данный класс равным <tex>\Sigma^*</tex> (программа, возвращающая результат функции ''random''(), подошла бы для любого языка).
 +
 
{{Определение
 
{{Определение
 
|definition=
 
|definition=
<tex>BPP_{weak}</tex> — множество языков <tex>L</tex>, для которых существует <tex>p</tex>, такая, что для любого <tex>x</tex>:
+
<tex>\mathrm{BPP_{weak}}</tex> — класс языков <tex>L</tex>, для которых существует такая ВМТ <tex>p</tex>, что для любого <tex>x</tex>:
#<tex>P(p(x)=[x \in L]) \ge \frac {1}{2} + \frac {1} {q(|x|)}</tex>, где <tex>q</tex>-полином;
+
#<tex>P(p(x)=[x \in L]) \ge \frac {1}{2} + \frac {1} {q(|x|)}</tex>, где <tex>q</tex>-полином и <tex>q(|x|) \ge 3</tex>;
#<tex>T(p(x)) \le poly(|x|)</tex> для любой вероятностной ленты.
+
#<tex>T(p, x) \le poly(|x|)</tex> для любой вероятностной ленты.
 
}}
 
}}
  
 
{{Определение
 
{{Определение
 
|definition=
 
|definition=
<tex>BPP_{strong}</tex> — множество языков <tex>L</tex>, для которых существует <tex>p</tex>, такая, что для любого <tex>x</tex>:
+
<tex>\mathrm{BPP_{strong}}</tex> — класс языков <tex>L</tex>, для которых существует такая ВМТ <tex>p</tex>, что для любого <tex>x</tex>:
#<tex>P(p(x)=[x \in L]) \ge 1 - 1 / {2^{q(|x|)}}</tex>, где <tex>q</tex>-полином;
+
#<tex>P(p(x)=[x \in L]) \ge 1 - \frac {1} {2^{q(|x|)}}</tex>, где <tex>q</tex>-полином и <tex>q(|x|) \ge 3</tex>;
#<tex>T(p(x)) \le poly(|x|)</tex> для любой вероятностной ленты.
+
#<tex>T(p, x) \le poly(|x|)</tex> для любой вероятностной ленты.
 
}}
 
}}
 +
 +
==Теорема==
 +
{{Теорема
 +
|statement= <tex>\mathrm{BPP} = \mathrm{BPP_{weak}} = \mathrm{BPP_{strong}}</tex>.
 +
|proof=
 +
В  доказательстве будет использоваться ''неравенство Чернова'': <br>
 +
<tex>\forall p :  \frac {1} {2} \le p \le 1: \sum\limits_{i = \lfloor \frac{n}{2} \rfloor + 1}^n \binom{n}{i}p^i (1 - p)^{n - i} \ge 1 - \mathrm{e}^{- 2n \left( {p - \frac{1}{2}} \right)^2}</tex>
 +
 +
 +
* Докажем, что <tex>\mathrm{BPP} = \mathrm{BPP_{weak}}</tex>
 +
# <tex>\mathrm{BPP} \subseteq \mathrm{BPP_{weak}}</tex> <br> Это следует из определений <tex>\mathrm{BPP}</tex> и <tex>\mathrm{BPP_{weak}}</tex>.
 +
# <tex>\mathrm{BPP_{weak}} \subseteq \mathrm{BPP}</tex> <br> Пусть <tex>L \in \mathrm{BPP_{weak}}</tex>. Тогда <tex>\exists p : P(p(x)=[x \in L]) \ge \frac {1}{2} + \frac {1} {q(|x|)}</tex>. <br> Построим ВМТ <tex>p_1</tex>, которая для входа <tex>x</tex> запускает <tex>p(x)</tex> <tex>n</tex> раз, и принимает <tex>x</tex>, если больше половины запусков принимают его. <br> Подберем <tex>n</tex>, такое, что <tex>P(p_1(x)=[x \in L]) \ge \frac {2}{3}</tex> и <tex>T(p_1(x)) \le poly(|x|)</tex>. <br> Вероятность <tex>P</tex> того, что <tex>p_1(x)</tex> даст правильный результат равна вероятности, что больше половины запусков <tex>p(x)</tex> дадут правильный результат. Тогда по схеме Бернулли <tex>P = \sum\limits_{i = \lfloor \frac{n}{2} \rfloor + 1}^n \binom{n}{i}p^i (1 - p)^{n - i}</tex>, где <tex>p=\frac {1}{2} + \frac {1} {q(|x|)}</tex> — вероятность, что запуск <tex>p(x)</tex> даст правильный ответ. По неравенству Чернова : <tex> P  \ge 1 - \mathrm{e}^{- 2n \left( {p - \frac{1}{2}} \right)^2} </tex>. То есть для того, чтобы <tex>P(p(x)=[x \in L]) \ge \frac {2}{3}</tex> достаточно подобрать такое <tex>n</tex>, что <tex>1 - \mathrm{e}^{- 2n \left( {p - \frac{1}{2}} \right)^2} \ge \frac {2}{3}</tex>. Получаем, что <tex>n \ge \frac {\ln 3} {2(p - \frac {1} {2})^2} = \frac {{q(|x|)}^2 \ln 3}{2} </tex>. Возьмем <tex>n = \lceil \frac {{q(|x|)}^2 \ln 3}{2} \rceil </tex>, тогда неравенство <tex>T(p_1(x)) \le poly(|x|)</tex> будет выполнено.
 +
 +
* Докажем, что <tex>\mathrm{BPP} = \mathrm{BPP_{strong}}</tex>
 +
# <tex>\mathrm{BPP_{strong}} \subseteq \mathrm{BPP} </tex> <br> Это следует из определений <tex>\mathrm{BPP}</tex> и <tex>\mathrm{BPP_{strong}}</tex>.
 +
# <tex>\mathrm{BPP} \subseteq \mathrm{BPP_{strong}}</tex> <br> Пусть <tex>L \in \mathrm{BPP}</tex>. Тогда <tex>\exists p : P(p(x)=[x \in L]) \ge \frac {2}{3}</tex>. <br> Построим ВМТ <tex>p_1</tex>, которая для входа <tex>x</tex> запускает <tex>p(x)</tex> <tex>n</tex> раз, и принимает <tex>x</tex>, если больше половины запусков принимают его. <br> Подберем <tex>n</tex>, такое, что <tex>P(p_1(x)=[x \in L]) \ge 1 - \frac {1}{2^{q(|x|)}}</tex> и <tex>T(p_1(x)) \le poly(|x|)</tex>. <br> Проводя рассуждения, аналогичные изложенным в доказательстве <tex>\mathrm{BPP_{weak}} \subseteq \mathrm{BPP}</tex>, получаем, что <tex>1 - \mathrm{e}^{- 2n \left( {p - \frac{1}{2}} \right)^2} \ge 1 - \frac {1}{2^{q(|x|)}}</tex>, где <tex>p = \frac {2} {3}</tex>. Отсюда <tex>n \ge \frac {{q(|x|)} \ln 2}{2({\frac {2}{3} - \frac {1}{2}})^2} </tex>. Возьмем <tex>n = \lceil 18 {q(|x|)} \ln 2 \rceil </tex>, тогда неравенство <tex>T(p_1(x)) \le poly(|x|)</tex> будет выполнено.
 +
}}
 +
 +
{{Теорема
 +
|statement =
 +
<tex>\mathrm{RP} \cup \mathrm{coRP} \subset \mathrm{BPP}</tex>.
 +
|proof =
 +
Пусть <tex>p</tex> — программа для <tex>L \in \mathrm{RP}</tex>. Программу <tex>q</tex> для <tex>\mathrm{BPP}</tex> определим следующим образом:
 +
  <tex>q</tex>(x)
 +
    u <- <tex>p</tex>(x)
 +
    v <- <tex>p</tex>(x)
 +
    '''return''' u '''or''' v
 +
Пусть <tex>x \in L</tex>. В этом случае вероятность ошибки равна <tex>\operatorname{P}(u = 0, v = 0) = \operatorname{P}(u = 0) \cdot \operatorname{P}(v = 0) \le 1/4</tex>.
 +
 +
Пусть <tex>x \notin L</tex>. Тогда с вероятностью <tex>1</tex> будет верно <tex>u = 0, v = 0</tex> и <tex>q</tex> вернет правильный ответ.
 +
 +
Аналогично доказывается, что <tex>\mathrm{coRP} \subset \mathrm{BPP}</tex>.
 +
}}
 +
 +
== См. также ==
 +
* [[Вероятностные вычисления. Вероятностная машина Тьюринга]] <br>
 +
* [http://en.wikipedia.org/wiki/Chernoff_bound Неравенство Чернова]
 +
 +
[[Категория:Классы сложности]]
 +
[[Категория:Теория формальных языков]]

Версия 16:03, 14 ноября 2018

Определения

Определение:
[math]\mathrm{BPP}[/math] (от bounded probabilistic polynomial) — множество языков [math]L[/math], для которых существует такая ВМТ [math]p[/math], что для любого [math]x[/math]:
  1. [math]P(p(x) = [x \in L]) \ge 2/3[/math];
  2. [math]T(p, x) \le poly(|x|)[/math] для любой вероятностной ленты.


[math]\mathrm{BPP}[/math] — сложностный класс, допускающий двусторонние ошибки. Константу [math]2/3[/math] можно заменить на любое число из промежутка [math](1/2, 1)[/math], так как требуемой вероятности можно добиться множественным запуском [math]p[/math]. Замена константы на [math]1/2[/math] сделала бы данный класс равным [math]\Sigma^*[/math] (программа, возвращающая результат функции random(), подошла бы для любого языка).


Определение:
[math]\mathrm{BPP_{weak}}[/math] — класс языков [math]L[/math], для которых существует такая ВМТ [math]p[/math], что для любого [math]x[/math]:
  1. [math]P(p(x)=[x \in L]) \ge \frac {1}{2} + \frac {1} {q(|x|)}[/math], где [math]q[/math]-полином и [math]q(|x|) \ge 3[/math];
  2. [math]T(p, x) \le poly(|x|)[/math] для любой вероятностной ленты.


Определение:
[math]\mathrm{BPP_{strong}}[/math] — класс языков [math]L[/math], для которых существует такая ВМТ [math]p[/math], что для любого [math]x[/math]:
  1. [math]P(p(x)=[x \in L]) \ge 1 - \frac {1} {2^{q(|x|)}}[/math], где [math]q[/math]-полином и [math]q(|x|) \ge 3[/math];
  2. [math]T(p, x) \le poly(|x|)[/math] для любой вероятностной ленты.


Теорема

Теорема:
[math]\mathrm{BPP} = \mathrm{BPP_{weak}} = \mathrm{BPP_{strong}}[/math].
Доказательство:
[math]\triangleright[/math]

В доказательстве будет использоваться неравенство Чернова:
[math]\forall p : \frac {1} {2} \le p \le 1: \sum\limits_{i = \lfloor \frac{n}{2} \rfloor + 1}^n \binom{n}{i}p^i (1 - p)^{n - i} \ge 1 - \mathrm{e}^{- 2n \left( {p - \frac{1}{2}} \right)^2}[/math]


  • Докажем, что [math]\mathrm{BPP} = \mathrm{BPP_{weak}}[/math]
  1. [math]\mathrm{BPP} \subseteq \mathrm{BPP_{weak}}[/math]
    Это следует из определений [math]\mathrm{BPP}[/math] и [math]\mathrm{BPP_{weak}}[/math].
  2. [math]\mathrm{BPP_{weak}} \subseteq \mathrm{BPP}[/math]
    Пусть [math]L \in \mathrm{BPP_{weak}}[/math]. Тогда [math]\exists p : P(p(x)=[x \in L]) \ge \frac {1}{2} + \frac {1} {q(|x|)}[/math].
    Построим ВМТ [math]p_1[/math], которая для входа [math]x[/math] запускает [math]p(x)[/math] [math]n[/math] раз, и принимает [math]x[/math], если больше половины запусков принимают его.
    Подберем [math]n[/math], такое, что [math]P(p_1(x)=[x \in L]) \ge \frac {2}{3}[/math] и [math]T(p_1(x)) \le poly(|x|)[/math].
    Вероятность [math]P[/math] того, что [math]p_1(x)[/math] даст правильный результат равна вероятности, что больше половины запусков [math]p(x)[/math] дадут правильный результат. Тогда по схеме Бернулли [math]P = \sum\limits_{i = \lfloor \frac{n}{2} \rfloor + 1}^n \binom{n}{i}p^i (1 - p)^{n - i}[/math], где [math]p=\frac {1}{2} + \frac {1} {q(|x|)}[/math] — вероятность, что запуск [math]p(x)[/math] даст правильный ответ. По неравенству Чернова : [math] P \ge 1 - \mathrm{e}^{- 2n \left( {p - \frac{1}{2}} \right)^2} [/math]. То есть для того, чтобы [math]P(p(x)=[x \in L]) \ge \frac {2}{3}[/math] достаточно подобрать такое [math]n[/math], что [math]1 - \mathrm{e}^{- 2n \left( {p - \frac{1}{2}} \right)^2} \ge \frac {2}{3}[/math]. Получаем, что [math]n \ge \frac {\ln 3} {2(p - \frac {1} {2})^2} = \frac {{q(|x|)}^2 \ln 3}{2} [/math]. Возьмем [math]n = \lceil \frac {{q(|x|)}^2 \ln 3}{2} \rceil [/math], тогда неравенство [math]T(p_1(x)) \le poly(|x|)[/math] будет выполнено.
  • Докажем, что [math]\mathrm{BPP} = \mathrm{BPP_{strong}}[/math]
  1. [math]\mathrm{BPP_{strong}} \subseteq \mathrm{BPP} [/math]
    Это следует из определений [math]\mathrm{BPP}[/math] и [math]\mathrm{BPP_{strong}}[/math].
  2. [math]\mathrm{BPP} \subseteq \mathrm{BPP_{strong}}[/math]
    Пусть [math]L \in \mathrm{BPP}[/math]. Тогда [math]\exists p : P(p(x)=[x \in L]) \ge \frac {2}{3}[/math].
    Построим ВМТ [math]p_1[/math], которая для входа [math]x[/math] запускает [math]p(x)[/math] [math]n[/math] раз, и принимает [math]x[/math], если больше половины запусков принимают его.
    Подберем [math]n[/math], такое, что [math]P(p_1(x)=[x \in L]) \ge 1 - \frac {1}{2^{q(|x|)}}[/math] и [math]T(p_1(x)) \le poly(|x|)[/math].
    Проводя рассуждения, аналогичные изложенным в доказательстве [math]\mathrm{BPP_{weak}} \subseteq \mathrm{BPP}[/math], получаем, что [math]1 - \mathrm{e}^{- 2n \left( {p - \frac{1}{2}} \right)^2} \ge 1 - \frac {1}{2^{q(|x|)}}[/math], где [math]p = \frac {2} {3}[/math]. Отсюда [math]n \ge \frac {{q(|x|)} \ln 2}{2({\frac {2}{3} - \frac {1}{2}})^2} [/math]. Возьмем [math]n = \lceil 18 {q(|x|)} \ln 2 \rceil [/math], тогда неравенство [math]T(p_1(x)) \le poly(|x|)[/math] будет выполнено.
[math]\triangleleft[/math]
Теорема:
[math]\mathrm{RP} \cup \mathrm{coRP} \subset \mathrm{BPP}[/math].
Доказательство:
[math]\triangleright[/math]

Пусть [math]p[/math] — программа для [math]L \in \mathrm{RP}[/math]. Программу [math]q[/math] для [math]\mathrm{BPP}[/math] определим следующим образом:

 [math]q[/math](x)
   u <- [math]p[/math](x)
   v <- [math]p[/math](x)
   return u or v

Пусть [math]x \in L[/math]. В этом случае вероятность ошибки равна [math]\operatorname{P}(u = 0, v = 0) = \operatorname{P}(u = 0) \cdot \operatorname{P}(v = 0) \le 1/4[/math].

Пусть [math]x \notin L[/math]. Тогда с вероятностью [math]1[/math] будет верно [math]u = 0, v = 0[/math] и [math]q[/math] вернет правильный ответ.

Аналогично доказывается, что [math]\mathrm{coRP} \subset \mathrm{BPP}[/math].
[math]\triangleleft[/math]

См. также